Telescopic sum issues, cant get Sk

This will give a new problem with a series that is equivalent to the original, but which doesn't have the 3s in it. This should make the problem easier to do.
  • #1
yuming
1
0
1. Homework Statement
Find a formula for the nth partial sum of the series and use it to find the series' sum if the series converges

Homework Equations


3/(1*2*3) + 3,/(2*3*4) + 3/(3*4*5) +...+ 3/n(n+1)(n+2)

The Attempt at a Solution


the first try, i tried using partial fraction which equals to A/(n) + B/(n+1) + C/(n+2). which made me get (3/2n - 3/n+1 + 3/2(n+2). Sn= (3/2 - 3/2 +3/6) + (3/4 - 1 + 3/8) + (3/6 - 3/4 + 3/10) + (3/8 - 3/5 + 3/12). i can't cancel it correctly. please help
i saw the correct working which the partial fraction suppose to be A/(n)(n+1) - B/(n+1)(n+2). didnt make sense to me because i thought in partial fraction we are suppose to split all the parts? and i tried that way and i would get A/(n)(n+1) + B/(n+1)(n+2) instead of A/(n)(n+1) - B/(n+1)(n+2). (couldnt get negative b) please help, have been trying to solve this for days.
 
Last edited by a moderator:
Physics news on Phys.org
  • #2
yuming said:
1. Homework Statement
Find a formula for the nth partial sum of the series and use it to find the series' sum if the series converges

Homework Equations


3/(1*2*3) + 3,/(2*3*4) + 3/(3*4*5) +...+ 3/n(n+1)(n+2)

The Attempt at a Solution


the first try, i tried using partial fraction which equals to A/(n) + B/(n+1) + C/(n+2). which made me get (3/2n - 3/n+1 + 3/2(n+2). Sn= (3/2 - 3/2 +3/6) + (3/4 - 1 + 3/8) + (3/6 - 3/4 + 3/10) + (3/8 - 3/5 + 3/12). i can't cancel it correctly. please help
i saw the correct working which the partial fraction suppose to be A/(n)(n+1) - B/(n+1)(n+2). didnt make sense to me because i thought in partial fraction we are suppose to split all the parts? and i tried that way and i would get A/(n)(n+1) + B/(n+1)(n+2) instead of A/(n)(n+1) - B/(n+1)(n+2). (couldnt get negative b) please help, have been trying to solve this for days.

Suggestions: (i) forget the '3' in the numerator; you can always put it back after you have done the summation; (ii) write ##\frac{1}{n(n+2)}## in partial fractions, then multiply by ##1/(n+1)## afterward.
 
  • #3
yuming said:
1. Homework Statement
Find a formula for the nth partial sum of the series and use it to find the series' sum if the series converges

Homework Equations


3/(1*2*3) + 3,/(2*3*4) + 3/(3*4*5) +...+ 3/n(n+1)(n+2)

The Attempt at a Solution


the first try, i tried using partial fraction which equals to A/(n) + B/(n+1) + C/(n+2). which made me get (3/2n - 3/n+1 + 3/2(n+2). Sn= (3/2 - 3/2 +3/6) + (3/4 - 1 + 3/8) + (3/6 - 3/4 + 3/10) + (3/8 - 3/5 + 3/12). i can't cancel it correctly. please help
First off, you need more parentheses. When you write 3/2n, that means ##\frac 3 2 n##, not ##\frac3 {2n}## as you intended. When you write 3/n + 1, that's even worse, as it means ##\frac 3 n + 1##
yuming said:
i saw the correct working which the partial fraction suppose to be A/(n)(n+1) - B/(n+1)(n+2). didnt make sense to me because i thought in partial fraction we are suppose to split all the parts? and i tried that way and i would get A/(n)(n+1) + B/(n+1)(n+2) instead of A/(n)(n+1) - B/(n+1)(n+2). (couldnt get negative b) please help, have been trying to solve this for days.
You can split it however you want. I wouldn't have thought of this approach, but working it through, it makes sense.

Decompose by setting ##\frac{1}{n(n + 1)(n + 2)} = \frac{A}{n(n + 1)} + \frac{B}{(n + 1)(n + 2)}##. After you find A and B, you will have a series that telescopes nicely.
Note: to simplify things you can take out a factor of 3 from all of the terms in the original problem.
 

What are telescopic sum issues?

Telescopic sum issues refer to problems encountered when trying to calculate the sum of an infinite sequence. This type of sum is often referred to as a telescoping sum because as you add more terms, the partial sums "telescope" or cancel out, leaving only a few terms that contribute to the overall sum.

What does "cant get Sk" mean?

"Cant get Sk" is a common notation used in telescopic sum problems. It refers to the partial sum of the first k terms in the infinite sequence. In other words, it represents the sum of the first k terms before the telescoping effect takes place.

How do I solve telescopic sum issues?

To solve telescopic sum issues, you need to identify the pattern in the sequence and use it to simplify the expression. This typically involves factoring and canceling out terms. Once you have simplified the expression, you can take the limit as k approaches infinity to find the sum of the infinite sequence.

What is the role of telescopic sums in mathematics?

Telescopic sums are important in mathematics because they allow us to calculate the sum of infinite sequences, which would otherwise be impossible. They also help us understand the concept of convergence and the behavior of infinite series.

Are there any tips for solving telescopic sum issues?

One useful tip for solving telescopic sum issues is to look for common factors in the terms of the sequence. This can help simplify the expression and make it easier to identify the pattern. Another tip is to try manipulating the sequence by adding or subtracting different terms to see if it helps to simplify the expression.

Similar threads

  • Calculus and Beyond Homework Help
Replies
1
Views
255
  • Calculus and Beyond Homework Help
Replies
1
Views
214
  • Calculus and Beyond Homework Help
Replies
12
Views
2K
  • Calculus and Beyond Homework Help
Replies
6
Views
234
  • Calculus and Beyond Homework Help
Replies
9
Views
927
  • Calculus and Beyond Homework Help
Replies
7
Views
956
  • Calculus and Beyond Homework Help
Replies
7
Views
1K
  • Calculus and Beyond Homework Help
Replies
8
Views
946
  • Calculus and Beyond Homework Help
Replies
7
Views
2K
  • Calculus and Beyond Homework Help
Replies
3
Views
413
Back
Top